[ 3 / biz / cgl / ck / diy / fa / ic / jp / lit / sci / vr / vt ] [ index / top / reports ] [ become a patron ] [ status ]
2023-11: Warosu is now out of extended maintenance.

/sci/ - Science & Math


View post   

File: 239 KB, 702x800, __remilia_scarlet_touhou_drawn_by_spacezin__ed17eeef456e54f348f711a1475accba.jpg [View same] [iqdb] [saucenao] [google]
15412116 No.15412116 [Reply] [Original]

Formerly >>15379528

Making this earlier since the previous thread refuses to die

>what is /sqt/ for?
Questions regarding maths and science. Also homework.
>where do I go for advice?
>>>/sci/scg or >>>/adv/
>where do I go for other questions and requests?
>>>/wsr/ >>>/g/sqt >>>/diy/sqt etc.
>how do I post math symbols (Latex)?
rentry.org/sci-latex-v1
>a plain google search didn't return anything, is there anything else I should try before asking the question here?
scholar.google.com
>where can I search for proofs?
proofwiki.org
>where can I look up if the question has already been asked here?
warosu.org/sci
eientei.xyz/sci
>how do I optimize an image losslessly?
trimage.org
pnggauntlet.com
>how do I find the source of an image?
images.google.com
tineye.com
saucenao.com
iqdb.org

>where can I get:
>books?
libgen.rs
annas-archive.org
stitz-zeager.com
openstax.org
activecalculus.org
>articles?
sci-hub.st
>book recs?
sites.google.com/site/scienceandmathguide
4chan-science.fandom.com/wiki//sci/_Wiki
math.ucr.edu/home/baez/physics/Administrivia/booklist.html
>online courses and lectures?
khanacademy.org
>charts?
imgur.com/a/pHfMGwE
imgur.com/a/ZZDVNk1
>tables, properties and material selection?
www.engineeringtoolbox.com
www.matweb.com
www.chemspider.com

Tips for asking questions here:
>attach an image (animal images are ideal, you can grab them from >>>/an/. Alternatively use anime from safebooru.donmai.us)
>avoid replying to yourself
>ask anonymously
>recheck the Latex before posting
>ignore shitpost replies
>avoid getting into arguments
>do not tell us where is it you came from
>do not mention how [other place] didn't answer your question so you're reposting it here
>if you need to ask for clarification fifteen times in a row, try to make the sequence easy to read through
>I'm not reading your handwriting
>I'm not flipping that sideways picture
>I'm not google translating your spanish
>don't ask to ask
>don't ask for a hint if you want a solution
>xyproblem.info

>> No.15412142
File: 449 KB, 595x842, __remilia_scarlet_and_flandre_scarlet_touhou_drawn_by_gunjou_row__30165c0fd52fd7fc12f7076216ff1f31.png [View same] [iqdb] [saucenao] [google]
15412142

Unanswered questions:

Maths questions:
>>15383448
>>15384307
>>15398880
>>15404915
>>15406337 ["crescent" isn't a synonym for increasing]
>>15411763

Biology questions:
>>15383262

Engineering questions:
>>15384403

Physics questions:
>>15386100
>>15411538

Stupid questions:
>>15380694
>>15389593
>>15390294
>>15390655
>>15404459
>>15408586

>> No.15412175

Best resource to trully learn quantum computing? (quantum algorithms etc.)
Are quantum mechanics necessary for quantum computing?

My CS uni doesn't have a fucking quantum computing course so ideally I should at least study the basics on my own. A good course suggestion will do (it is probably preferable to reading a 1000-paged textbook).
I don't want it to be codemonkey-ish like the ones google suggests, I want it to be solid and math-based. I am strong in mathematics for an undergrad (but I don't know if I'll be able to understand something which is written for graduate math students)

>> No.15412182
File: 229 KB, 700x700, gift.png [View same] [iqdb] [saucenao] [google]
15412182

>>15412175
forgot to post pic.
take is as a thank you for anyone who will care to answer.

>> No.15412194

What is the relation between integrals and bivectors?
My gut keeps telling me there's some cool relation between the two, like effectively taking integrals with simple matrix multiplication, but I can't figure out what

>> No.15412203

>>15412194
I understand the words but not the way you string them together

>> No.15412564

>>15412203
Me neither. All I know is that adding bivectors by adding their areas, and subtracting areas if the bivector turns the opposite way sounds a hell of a lot like how taking integrals work.

Everyone's seen that insane video that explains how the geometric product of two vectors is basically Euler's formula. There's some other crazy stuff like Cauchy's integal theorem out there that makes it seem like there's some hidden relation somewhere that somebody's looked into this at least once before, that relates vector multiplication with integrals.

>> No.15412566

>>15412116
why do you jack off to children's cartoons? is it because you're a pedophile?

>> No.15412613
File: 114 KB, 943x940, __matara_okina_touhou_drawn_by_jill_07km__4553718e7aa11fe3ab8709b93fe5c49d.jpg [View same] [iqdb] [saucenao] [google]
15412613

>>15412566
I missed you bro, it's good to see you again.
>>15412194
If you consider that integrals and covectors are, in a sense, dual, since you can evaluate them against each other to get a number, then you can consider the duality between vectors and covectors to be an infinitesimal version of that, and then bivectors are infinitesimal area integrals.
But I don't think this is useful for anything or even a helpful analogy to make.

>> No.15412885

Any place to find market data for chemical products for free?

>> No.15412951
File: 5 KB, 692x298, Untitled.png [View same] [iqdb] [saucenao] [google]
15412951

Consider the spaces X and Y (see pic). Both of them can be thought of as the closed unit disk D from which an open disk was removed: In X's case, we remove an open disk whose boundary is tangent to the boundary of D at a single point; in Y's case, we remove an open disk from the interior of D and obtain a closed annulus.

Are these spaces homeomorphic?

>> No.15413092

>>15412175
Preskill's quantum computing notes are still pretty good

>> No.15413120

>>15412951
is X still a connected space at (the limit on) the discs' tangency?

>> No.15413133

>>15412951
if tangent than no

>> No.15413162

>>15412175
GTM has a quantum book that requires undergraduate analysis.

>> No.15413597

Hey anons, any idea how to figure out how far a particulate moving at relativistic speeds could move through the atmosphere before burning up?
Nothing rocket science proof, just a fermi estimate kinda thing. I'm thinking if I can calculate the friction I can calculate how quickly it'll get vaporized/explode in a fireball.

>> No.15413790
File: 89 KB, 600x600, cosmic_ray.jpg [View same] [iqdb] [saucenao] [google]
15413790

>>15413597
that's extremely non-trivial. it has nothing to do with friction and instead would required knowing particle cross sections, velocity, air density and composition as a function of height etc ...

> it'll get vaporized/explode in a fireball
that isn't what happens. you get a particle collision / decay similar to what you'd get inside a particle accelerator.

>> No.15413797

>>15413790
To clarify, I am considering a particulate of bulk material. A tiny particulate, in the order of the micrometer or nanometer, but still made up of a large amount of individual atoms.
Though I am sure this doesn't make it any more trivial, likely less so.
Then, say I set all those parameters. Infinite flat plane in a uniform atmosphere of pure gas of your preference at 100 kPa, 300 K. Where would I begin to develop a solution? I don't mind doing the footwork, just not sure where to start or what textbook would consider this kind of scenario.

>> No.15413807

CPT symmetry
In simple terms, what does T symmetry apply to exactly, antiparticles?
Can you describe what it actually means and looks like in practice?
Does it mean antiparticles would have entropy in reverse? I don't get it.
You can't tell me on a hypothetical Antimatter planet that an object made of antimatter is going backwards through time right? So in layman's terms what would actually happen to a thing that obeys T symmetry?

>> No.15413820

>>15413797
I wouldn't know where to start t.b.h. The usual air resistance or thermodynamic equations wouldn't apply at supersonic or relativistic speeds. I would suggest looking at NASA resources, they have a surprising amount of technical articles about such things if you look hard enough.

>> No.15413829

>>15413820
Yeah, I had the thought that the closest situation that had any chance to have been seriously investigated would be some aerospace folks considering the effects of a ship traveling through the interstellar medium at relativistic speeds. I'll try digging in that direction.
If I had to come up with an instinctive simple analytical solution, it would be to consider each collision between an air molecule and the pellet its own (high energy) event, consider how many collisions there are in the pellet's path, and how each collision affects its moment/thermodynamic status.

>> No.15413830

>>15413807
CPT symmetry isn't about the particles, it is about symmetries of the equations we use to describe them. It says is you reverse the sign of the Charge, the Parity, and the Time parameter of the equation describing a particle you get an equation that describes its anti-particle. It doesn't mean anti-particles are going backwards in time.

> So in layman's terms what would actually happen to a thing that obeys T symmetry?
Nothing. All the laws of physics obey T symmetry and all it means is we can use them to calculate the state of a system in the past and in the future.

>> No.15413833
File: 166 KB, 890x661, wut da zog?.png [View same] [iqdb] [saucenao] [google]
15413833

why mirroring only worky on x axis but no y axis??

>> No.15413846

>>15413833
That is a false statement.

>> No.15413880

what is a good first textbook on AI and/or machine learning for someone with an EE/controls + signal processing degree

>> No.15414275

>>15413833
the left-right axis stays the same, it's the front-back axis that gets flipped

>> No.15414323

>>15413833
https://youtube.com/watch?v=o_D-HsMOYQ8

>> No.15414360 [DELETED] 

Just started teaching myself linear algebra to be exact (also not sure about the exact terms in English, hope this makes sense).

Assuming the domain are the natural numbers including zero and the codomain are the integers.
f(x) = (-1)^x * 2x + (1/2) * ((-1)^(x) - 1)

How do I check if this is surjective / injective or not? If I insert f(1) I get -3 and with f(2) I get 4 but this doesn't seem enough for a proof.

>> No.15414382

Just started teaching myself linear algebra to be exact (also not sure about the exact terms in English, hope this makes sense).

Assuming the domain are the natural numbers including zero and the codomain are the integers.
f(x) = (-1)^(x) * 2x + (1/2) * ((-1)^(x) - 1)

How do I check if this is surjective / injective or not? If I insert f(1) I get -3 and with f(2) I get 4 but this doesn't seem enough for a proof.

>> No.15414493

I'm going through Luenberger's Optimization by Vector Space Methods and I don't understand one of the definitions (p.17).
>Let S be a nonempty subset of a vector space X. The LINEAR VARIETY GENERATED BY S denoted v(S) is defined as the intersection of all linear varieties in X that contain S.
Isn't the intersection of all linear varieties that contain S just S itself?

>> No.15414508

>>15414382
Assume n to be any integer, find a nonnegative integer x in terms of n that that maps to it. This shows surjectivity. Assume x and y to be any nonnegative integer such that their images are the same. Show that x must equal y. This shows injectivity.

>> No.15414532
File: 552 KB, 1464x1503, __reisen_udongein_inaba_touhou_drawn_by_temmie_chang__bdf79162b32c23610a83211f3d4c7882.png [View same] [iqdb] [saucenao] [google]
15414532

>>15414493
>Isn't the intersection of all linear varieties that contain S just S itself?
Why would it be? Note that S is a subset, not a subspace. It can be something random like [math] \{ (0, 0), (0, 1) \} \in \mathbb{R}^2 [/math]

>> No.15414537

>>15414493
It's the smallest linear variety that contains all of S. It's only equal to S if S is a linear variety.

>> No.15414542

>>15414508
>Assume n to be any integer, find a nonnegative integer x in terms of n that that maps to it. This shows surjectivity
Ok so like, let n = 2 then I try to solve 2 = (-1)^(x) * 2x + (1/2) * ((-1)^(x) - 1), right? But I feel like it's irrelevant what n is because I can't solve the right part no matter what? I'm pretty sure it's not surjective then but I don't feel like I am really able to prove it and think I'm doing something wrong while trying to do the equation.
>Assume x and y to be any nonnegative integer such that their images are the same. Show that x must equal y. This shows injectivity.
Ok so f(x) = f(y). But how do I know what to insert? If I take any nonnegative integer, like 2 and 3, f(2) = f(3) doesn't work, it's 4 = -7.

>> No.15414564
File: 322 KB, 2048x2048, __yakumo_yukari_touhou_drawn_by_shindiyue__04d247222c229bf5dd279d2bc34131cc.jpg [View same] [iqdb] [saucenao] [google]
15414564

>>15414382
>How do I check if this is surjective / injective or not?
It's not surjective.
As you've pointed out, [math]f(0) = 0, f(1) = 3, f(2) = 4[/math]. If [math]n > 2[/math] then [math]|f(n)| = \left| (-1)^n 2n + \dfrac{(-1)^n - 1}{2} \right| \geq 2n - 1 > 3[/math], so [math]2[/math] isn't in the image.

I'm honestly not sure if you were supposed to figure this out or if there's a mistake in the formula.

>> No.15414569

why are there so many women in astrophysics?

>> No.15414590

>>15414382
First rewrite [math]f(x)[/math] a bit so that it is easier to see what it does

[eqn]f(x) = (-1)^x \cdot 2x + \frac{(-1)^x - 1}{2} = \begin{cases} 2x & \text{for } x \equiv 0 \text{ (mod 2)} \\ -2x - 1 & \text{for } x \equiv 1 \text{ (mod 2)} \end{cases}
[/eqn]

Written like you immediately see that 1 is not in the image so it's not surjective.
To see that it is injective you can just find a left inverse like

[eqn]g(x) =\left \lceil \frac{|x| - 1}{2} \right \rceil [/eqn]

>> No.15414591

>>15414542
>Ok so like, let n = 2
No, you are assuming n to be 2. Assume n to be any integer, and show that it must always have a preimage x that corresponds to it. In this case, you would need to separate the cases of x being even and odd, to be able to solve it. If you want to disprove, you just need to come up with a counterxample i.e., find an integer such that no nonnegative integers could possibly map to it.

>But how do I know what to insert?
You are not supposed to insert. You are supposed to show if there exists two numbers x and y whose images are the same, it must follow they are the same. To disprove, you can find a counterexample i.e, find two distinct numbers with the same image.

You are supposed to work with variables not constants.

>> No.15414626 [DELETED] 

>>15414532
>>15414537
Yes, I think I just misunderstood the definition of a linear variety. For a circle in [math]\mathbb{R}^3[\math], the smallest linear variety encompassing it is a plane running along it. But I thought something like a cylinder encompassing the circle was also a linear variety. So I was imagining every possible shape that could encompass the circle. But a cylinder or any other solid is clearly not a linear variety now that I recall the definition.

>> No.15414652

>>15414532
>>15414537
Yes, I think I just misunderstood the definition of a linear variety. For a circle in [math]\mathbb{R}^3[/math], the smallest linear variety encompassing it is a plane running along it. But I thought something like a cylinder encompassing the circle was also a linear variety. So I was imagining every possible shape that could encompass the circle. But a cylinder or any other solid is clearly not a linear variety now that I recall the definition.

>> No.15415012
File: 309 KB, 1852x748, Screenshot 2023-05-03 at 6.32.16 PM.png [View same] [iqdb] [saucenao] [google]
15415012

>>15414652
Why are we allowed to take the factosr sin(x) and cos(x) out of their limits?? I'm assuming that is what is happening. Does this follow from the product rule? I've looked at like four separate textbooks and no one seems to explain this.

I've seen it explained that its because those factors don't depend on the limit so its okay to do but that explanation doesn't really explain much to me.

>> No.15415013

>>15415012
>>15414652
Sorry, didn't mean to reply to you

>> No.15415014

>>15415012
sin(x) and cos(x) are just constants. They are completely unrelated to h so you can pull them out of limits, just like how you can take a factor of 2 out for instance.

>> No.15415035

>>15415014
They don't look like constants to me, their value changes with x

>> No.15415044

>>15415035
Yes.
But you're not changing the value of x when you're taking the derivative at that specific x.

>> No.15415414

is there a symbol to denote an arbitrary operator like
>x ? y
and ? can be + or * or even some complex algorithm it doesn't matter.
f(x) is the closest I can think of.

>> No.15415423

What does the moment of a vector represent intuitively?
For example the moment of the momentum or the moment of a force.
I know how it's defined (which would be vector product between the distance vector of the chosen pole to the application point of the vector, and the vector)

>> No.15415436

>>15415414
[math] x*y \\ x \circ y \\ x \mathbin \square y \\ x \bullet y \\ x \mathrel \triangle y [/math]
etc.
If you are writing in [math] \mathrm \LaTeX[/math], you should add a [math] \texttt{\mathbin}[/math] before it.

>> No.15415604

>>15412116
Stupid question but how long does it takes for you to finish a math book? Do you work on exercise? Lastly, anyone here has experience with Zorich? If so, how was it, especially for exercise?

>> No.15415724

>>15415604
>Stupid question but how long does it takes for you to finish a math book?
Depends on the book.
>Do you work on exercise?
Obviously.
>Lastly, anyone here has experience with Zorich? If so, how was it, especially for exercise?
As a first course or even a second course, it's pretty awful and boring, especially because of the pedagogically unsound exercises. Baby Rudin is eternal king because of the exercises (and incomplete proofs which act like exercises). Abbott also has very good exercises, probably the best in any book I have read, but it is quite elementary.
Zorich is fine as a comprehensive reference text, and for its problems.

>> No.15415730

How to cheat on my calc 2 final? I desperately want to pass and be done with school but I really fucked up when I got sick earlier this semester

>> No.15415731
File: 32 KB, 669x724, notebook-demo.png [View same] [iqdb] [saucenao] [google]
15415731

I hate R so much it's unreal. It seems to have a more complete set of packages for my area of study than Python though.

I hate Rstudio I'd much rather use Jupyter notebooks. I am tempted to see if using R in Jupyter lessens my distaste for R. I'm not sure about learning R Markdown it just seems less enjoyable.

Does anyone have any experiences with jupyter R vs Rmarkdown

>> No.15415779
File: 4 KB, 259x194, projective.jpg [View same] [iqdb] [saucenao] [google]
15415779

How to construct a cellular complex structure on complex projective space [math]\mathbb{CP}^n[/math] such that real projective space [math]\mathbb{RP}^n[/math] becomes a sub-complex? (The standard cell decompositions of these spaces won't cut it.)

There's a suggested construction on MSE 3494652, but I'm having a super hard time understanding why it works and how the attaching maps are defined.

>> No.15415807
File: 22 KB, 952x406, duda.png [View same] [iqdb] [saucenao] [google]
15415807

Going through the Weissbluth, how is this step justified?(Chapter 13)
The [math]\psi^i[math] aren't ortogonal, just normalized. The [math]\ket n[math] are orthonormal and make a complete set, btw-

>> No.15415891
File: 298 KB, 1146x1500, 1667982993148819.jpg [View same] [iqdb] [saucenao] [google]
15415891

Scientifically speaking, what is the optimal way to start your day?

>> No.15416001

Is there a way to prove that given an integer a and a natural number b, a - b < a ?

>> No.15416018

>>15415731
these meme notebooks always seemed completely pointless to me
Rstudio is gay for sure

>> No.15416025

>>15416001
subtract a from both sides

>> No.15416109

>>15415891
>camp pinewood
based

>> No.15416157

>>15415012
>Why are we allowed to take the factosr sin(x) and cos(x) out of their limits??
In general, if the limit of h->L A(h) and limit of h->L B(h) both exist and are not infinite, then:

(limit of h->L (A(h) + B(h)) = (limit of h->L A(h)) + (limit of h->L B(h))
(limit of h->L (A(h) - B(h)) = (limit of h->L A(h)) - (limit of h->L B(h))
(limit of h->L (A(h) * B(h)) = (limit of h->L A(h)) * (limit of h->L B(h))
(limit of h->L (A(h) / B(h)) = (limit of h->L A(h)) / (limit of h->L B(h)) as long as (limit of h->L B(h)) is not zero.

This is the case because addition, subtraction, and multiplication are all continuous everywhere, and division is continuous as long as the divisor is nonzero.

They use that in your example to do the following derivation:
limit of h->0 [ (sin x cos h + cos x sin h - sin h) / h ]
= (factoring)
limit of h->0 [sin x * ((cos h - 1) / h) + cos x * (sin h / h) ]
= (limit of sum equals sum of limits)
limit of h->0 [sin x * ((cos h - 1) / h) ] + limit of h->0 [ cos x * (sin h / h) ]
= (limit of product equals product of limits, twice)
limit of h->0 [sin x] * limit of h->0 [ (cos h - 1) / h ] + limit of h->0 [cos x] * limit of h->0 [ sin h / h ]
= (limit body is independent of h, twice)
[sin x] * limit of h->0 [ (cos h - 1) / h ] + [cos x] * limit of h->0 [ sin h / h ]

Most arithmetic operations are continuous, and thus can be taken outside of a limit, *under certain conditions*. Addition, subtraction, and multiplication are continuous everywhere, and thus can always be taken outside a limit (as long as those limits exist and are finite); division is continuous everywhere except where the divisor is zero; sin() and cos() are continuous everywhere, but tan() has a number of exceptions; logarithms are continuous everywhere (in the real numbers, anyway); for exponentiation, the conditions are bit more complicated; etc.

>> No.15416237

>>15415724
Thank you anon. Honestly, I'm taking an analysis course, and been doing exercise from Fitzpatrick Advanced Calculus. The next course will cover similar topics (maybe a bit on integration) but we will transition to rudin. So, should I skip that course and just move ahead with topology?

>> No.15416258

>>15413807
>Can you describe what it actually means and looks like in practice?
An antiparticle acts just like an ordinary particle but the sign of any charges (e.g. electric charge) are flipped, that's all. T symmetry acts in the same mundane way on both particles and antiparticles. It flips the sign of the velocity and the spin. (If you are thinking about QM it might do some fussy things to the phase of the state too)

>Does it mean antiparticles would have entropy in reverse?
I understand you want to think about the arrow of time but T symmetry has nothing to do with entropy.

>> No.15416279

>>15412564
>this thing seems like this other thing
I strongly suspect that a categorification of each thing (bivectors and integrals) would "explain" this similarity. As others have said, that doesn't mean the observation is useful.

>> No.15416289

>>15415891
With a giant glass of mineralized water.
We are electric beings after all. Poor hydration limits brain function.

>> No.15416460

>>15416237
You can skip to Topology if you have studied metric spaces, compact spaces, connected sets, etc., in context of calculus. Otherwise, Topology is going to seem extremely unmotivated. Personally, I'd suggest solving at least the first 8 chapters of Baby Rudin before attempting Topology.

>> No.15416465
File: 53 KB, 1220x600, __inaba_tewi_touhou_drawn_by_m_m073111__f7b3a5c6b21e29b17eae9dde0fb26ee7.jpg [View same] [iqdb] [saucenao] [google]
15416465

>>15416109
Is Camp Pinewood any good now? Last time I played it a couple years ago it had a bunch of images but no actual content.

>> No.15416897

How do I solve [math] y' = y(xy^{7} -1)[/math] using substitution?

>> No.15416939
File: 421 KB, 620x616, 1668451695523149.png [View same] [iqdb] [saucenao] [google]
15416939

>>15416109
Uhh that's sherwood anon
>>15416465
First one got abandoned 2 years ago and they started remaking it.
Lost some soul but seems to be chugging along

>> No.15416966

>>15416897
That's a Bernoulli equation so you would probably use a substitution [math]u = y^{1 - \alpha}[/math] where in this case alpha is 8. That should give you a linear equation in u. Go lookup the Bernoulli Method.

>> No.15417029

>>15416939
>Uhh that's sherwood anon
yea my b, totally fucked that up. last time i read it was in high school. what a long strange trip its been.

>> No.15417052

What's an intuitive explanation on the function dot product? I want to be able to understand the Laplace transform with that

>> No.15417054

There is this kid from my university (a few years lower) who I know and is kind of a bad student. I just noticed a post on leddit asking for help (anonymously) with a very particular set of questions and I am 100% sure it is him. Do I help him out again?

>> No.15417117

In what point of the curve x + y +(y*x)^1/2 = -1 , the tangent line is parallel to the x axis?

>> No.15417148

>>15417117
> the tangent line is parallel to the x axis
Think about what this means, this only happens when the gradient is zero. How would you find the gradient of a curve?

>> No.15417167

>>15417148
derivative?

>> No.15417168

>>15417167
bingo! solve for dy/dx = 0

>> No.15417197

>>15417054
Whats the question?

>> No.15417198

>>15417117
hint: you may want to google/review "implicit differentiation" to solve this

>> No.15417241

No one could ever answer me this: if im 10 light years away from earth, look at earth, with a theoretical telescope such that i could see people, i would see them as they were 10 years ago (2013).
Now while looking at earth, i fly towards it at 0.5c, i.e. it takes 20 years to get there, so i arrive in 2043,
So i spent 20 years watching earth, but i start seeing it in 2013 and end in 2043 so i see a timespan of 30 years. So will i see everything at 133% speed? E.g. if i could make out people, would it look to me like a movie on fast forward?

>> No.15417255

>>15417241
You are swapping inertial frames in your problem description without realizing it.
If you are a planet keeping the same distance from Earth, and you are 10 ly away, you will indeed see Earth as it was 10 years ago.
Now when you jump in your spaceship and accelerate to 0.5c, you change inertial frames because, from your perspective, the distance to Earth compresses due to length contraction by a factor of γ≈1.15. Similarly, you now witness time dilation that actually slows down the apparent happenings on Earth by that same factor. Since the trip took 20 years proper time, the amount of time you see pass on Earth during your trip is only 20/1.15=17.4 years.
Use the Lorentz Transformation to find these values more easily.

>> No.15417260

>>15417241
0.5c isn't particularly fast (relatively speaking), so the effect of time dilation have a gamma factor of 1.15, so only a 15% change. But to answer your question yes, if you were flying towards Earth you would see everything apparently moving 15% faster because you are seeing all the light from Earth "sooner", if you flew away from Earth everything would appear to move 15% slower because it would take longer for the light to reach you.

>> No.15417319

Can anyone please post their solution homework in numerical analysis for ODE as well as integrals, because I am with 100% certainty failing this fucking course as I have no idea how to practically use the knowledge I've "gained". Thank you.

>> No.15417439 [DELETED] 

>>15416460
This is a great advice. Now I can't help but throw my life problem here. I should really be asking this on /adv/ but here is my situation. I'm retarded when it comes to injective and bijective proofs, like I can't come up with a proof for Schroder Bernstein theorem by myself. However, I liked to think I'm doing fine with real analysis like convergence (we are going to convergence and analysis of sequences of function). I'm in my senior year of undergrad (not math major) and I only have this summer to take as many classes as humanly possible as I'm graduating. In summer, they offer Linear Algebra (1-7 Chapters of Axler), Abstract Algebra (2, 5, 6 of Artin), Topology (1-3 chapters of Munkres) and Real Analysis (1-4 chapters of Baby Rudin), and Complex Analysis (Convergence in C, Analytic Functions, Power Series Expansion). What should I take to prepare myself (mathematical maturity) to tackle book like Zorich and the like (or eventually folland)? I want to join research in reinforcement learning but "an advisor" told me to learn as much math as possible before joining, so here we are. So, what should I take?

>> No.15417464

>>15416460
This is a great advice. Now I can't help but throw my life problem here. I should really be asking this on /adv/ but here is my situation. I'm retarded when it comes to injective and bijective proofs, like I can't come up with a proof for Schroder Bernstein theorem by myself. However, I liked to think I'm doing fine with real analysis like convergence (we are going to convergence and analysis of sequences of function). I'm in my senior year of undergrad (not math major) and I only have this summer to take as many classes as humanly possible as I'm graduating. In summer, they offer Linear Algebra (1-7 Chapters of Axler), Abstract Algebra (2, 5, 6 of Artin), Topology (1-3 chapters of Munkres) and Real Analysis (1-4 chapters of Baby Rudin), and Complex Analysis (Convergence in C, Analytic Functions, Power Series Expansion). What should I take to prepare myself (mathematical maturity) to tackle book like Zorich and the like (or eventually folland)? I want to join research in reinforcement learning but my advisor told me to learn as much math as possible before joining. So, what should I take?

>> No.15417474

>>15417439
You should first read a proof book like Vellman's first. You can read pretty much any undergraduate book after that. But of course, still there are going to be books that are going to be hard more so because they are boring like Zorich.

Pretty much all the subjects you listed is going to be unmotivated without Real Analysis. But the best would probably be Munkres' Topology since not the subject, but the book is more motivated than Rudin. In fact, the material in the first 3 chapters are essential for Rudin anyway.

Linear algebra does not require Analysis, but Axler provides no motivation nor application. I don't know what reinforcement learning is but since it has to do with machine learning, linear algebra would be quite important to it. However, since Axler talks nothing about matrices, I wouldn't recommend it. If you are willing to use a different book, you can use Rao & Bhimasankaram which is a LA book for statisticians. It talks about applications and focuses on matrices, but still rigorous. You'd need to connect the dots to your course syllabus though. Axler works directly with linear transformations, while R&B works with matrices, which are representation of linear transformations.

Algebra is a very good subject and will make you good at proofs, and I have heard while Artin is weird, it is motivated. However, it's hard to appreciate Algebra if you have not studied math for a while. It's all about connecting different fields of math.

So I would say: read Vellman first. Solve the entire book. Then do either:
Linear algebra from R&B or Topology from Munkres.

>> No.15417525

>>15417474
I guess will review Book of Proof or Vellman before summer then. I'm going to go for algebra and topology this summer. Thank you anon for your advice.

>> No.15417602

I just solved the Friedmann equation for a radiation-dominated universe. I have an expression for the scale factor now (depending on time). It goes like [math]\sqrt{A - Bt^2}[/math] so the universe expands first and then collapses again.

How am I supposed to find the maximal size of the universe with this? If I understand correctly the scaling factor is dimensionless, and I haven't been given a starting size, so I'm not sure how you could find a length.

>> No.15417632
File: 1.13 MB, 1198x1802, killme.png [View same] [iqdb] [saucenao] [google]
15417632

So I am reading through a textbook and I do not understand why one compound is used over another because I did not study material engineering, are there any semi-accessible textbooks on this?

>> No.15417640

>>15417602
>dimensionless
Proportional?

>> No.15417643

>>15417602
Are you sure that's correct? A radiation dominated universe shouldn't have enough energy density to eventually collapse and would have [math]a \propto \sqrt{t}[/math].

>> No.15417788

What kind of object is [math] \sqrt{z} [/math] strictly speaking?
For real, positive numbers it is a function. But for complex numbers it has more than one solution which fails the definition of a function. Or is the codomain assumed to be set of sets, to satisfy the condition of root being a function?

>> No.15417808

>>15417640
No, I mean it's a dimensionless quantity

>>15417643
I'm pretty sure yeah. It matches what my professor's lecture notes said and what they wanted me to find in the question.

>> No.15417807

>>15417260
>>15417255
Thank you big brain anons, idk why no one could ever explain this to me

>> No.15417811

>>15417788
it's a multivalued function, which is a fancy way of saying it acts like a perfectly normal function locally even if it isn't one globally

>> No.15417814

>>15417319
Idk if this helps but "houston math prep" on yt has probably the best videos out there on ODEs

>> No.15417861
File: 204 KB, 2000x2000, R.jpg [View same] [iqdb] [saucenao] [google]
15417861

Is there any way to make these suck less? Besides redesigning that garbage element of course.
Wouldn't it make more sense to have a high-conductivity element, and some sort of temperature-feedback loop, so an initial heating surge heats your pans right away and then lowers power to maintain heat?

>> No.15417879

>>15417861
theres regulations on how much power the elements can draw, and they usually max it out. nothing to be done.

>> No.15417909

>>15417879
I'm reading that high load residential circuits can provide up to 30-50 amps, which is about 3kW on 110V, and that many gas burners output 2-3kW. How isn't that in range?

>> No.15417924
File: 669 KB, 1155x1139, __remilia_scarlet_and_tatara_kogasa_touhou_drawn_by_rokugou_daisuke__a18280e14672c395f3dfba6ec7dcce22.png [View same] [iqdb] [saucenao] [google]
15417924

>>15416939
>First one got abandoned 2 years ago and they started remaking it.
>Lost some soul but seems to be chugging along
I see, I'll let it remain in the oven for some more then.

>> No.15417984
File: 51 KB, 1001x255, file.png [View same] [iqdb] [saucenao] [google]
15417984

>>15412116
I've started working through an undergrad LinAlg textbook because I've been forced to realize just how lacking my math knowledge had become after I stumbled through the mandatory courses and then never touched anything math related again.
Unfortunately, I've been filtered already by pic rel, and this exercise is skipped in the solutions manual.
I suspect that a proof would go something like this:
>If a function can be inversed, it must be bijective, and if its bijective, each value in the domain must correspond to one and only one value in the codomain and therefore it doesn't matter if you inverse it or take the intersection first.
But how the fuck do I write this in "mathematical notation" like you would see in a paper?

>> No.15417987
File: 28 KB, 431x477, file.png [View same] [iqdb] [saucenao] [google]
15417987

>>15417984
I drew a picture to help me visualize it better.

>> No.15417996

>>15417984
All functions have inverse images. Bijections have unique inverse images. You should read Vellman's How to Prove It.

>> No.15417997

>>15417984
Firstly inverse images and inverse functions are different things despite both being written as [math]f^{-1} [/math].
If [math]f[/math] goes [math]X \to Y[/math] then the inverse function if it exists goes [math]Y \to X[/math]. The inverse image on the other hand goes [math]2^Y \to 2^X[/math]. Here [math]2^X[/math] denotes the powerset of [math]X[/math]. That is the set of all subsets of [math]X[/math].


You will have to prove that
[eqn]f^{-1}(A \cap B) = \{x \in X | f(x) \in A \cap B \}[/eqn]
is equal to
[eqn]f^{-1}(A) \cap f^{-1} (B)= \{x \in X | f(x) \in A\} \cap \{x \in X | f(x) \in B\}[/eqn]

To prove that two sets are equal you have to show that they contain the same elements. So take an element of the first set and show it must be in the second set and then do the opposite direction.

>> No.15418001

where can I buy an actual, high-quality slide rule in this day and age

>> No.15418004

>>15417984
Images and inverse images are essentially sets, so very often the easiest and most straightforward way to prove equality of sets is to show mutual inclusion, that is
[math] f^{-1}(A \cap B) \subset f^{-1}(A) \cap f^{-1}(B) [/math]
and the other way around

>> No.15418232
File: 22 KB, 1000x600, Triangle mesh.png [View same] [iqdb] [saucenao] [google]
15418232

Is there a mathematical term for a triangle in a triangle mesh that only connects to one other triangle? For example the red triangle in the image.

>> No.15418347

I need lectures for linear algebra and mechanics, is there anything up on youtube that could be recommended ?

>> No.15418373

>>15418232
I would think of a vertex in a graph that has degree equal to 2.

>> No.15418442

>>15412116
My understanding is that random number generators are seeded. Does this mean that the first term of a randomly generated string of 256 bits will never be all zeros or all ones? Like never, not just 1/2^256 unlikely.

>> No.15418459

>>15418442
Pseudo RNG algorithms produce a fixed list of numbers that has all the properties of being completely random. The seed is simply the position in the list you start from. So in theory all numbers should be equally likely, be that all zeros or all ones.

>> No.15418470

>>15418459
But if the randomness is that the seed randomly selects a position in the algorithm list, how is that any different than randomly selecting a position in an ordered list?

>> No.15418475

>>15418470
The seed doesn't randomly select a position, it's the list that is random. If you use the same seed you always start from the same place in the list, i.e. you get the same "random" number back. Then the seed value is adjusted internally to pick the next list value on the next function call.

>> No.15418478

>>15418442
The seed comes from some 'random' source outside the algorithm. Often the time on the system clock is used

>> No.15418488

>>15418475
But then how is the fixed random list generated? Also with an earlier fixed random list?

>> No.15418498

>>15418488
That is the magic of which ever particular algorithm you use and they often rely on some advanced mathematics to do so. For example a common algorithm used in many programming languages is based on https://en.wikipedia.org/wiki/Mersenne_Twister

These algorithms produce streams of data that passed all the usual statistical checks for randomness, if you knew no better and didn't know the source you think it was perfectly random. However they aren't, they fake it. Hence the name pseudo random number generators.

>> No.15418534

>>15418498
It seems so precarious, though.

>> No.15418560

>>15418534
It good enough for most general purposes. Very few pieces of software require true absolute randomness. One of the situations where such algorithms are not good enough is encryption keys. They need an extra source of randomness to avoid being predictable - CloudFlare has a meme solution that actually works https://blog.cloudflare.com/randomness-101-lavarand-in-production/

>> No.15418598

Suppose that you started moving at origin 0 at a speed of 8 and halved your speed every "second" (Or whatever arbitrary amount of time). So at 0 seconds you would be at (0). At 1 second you would travel 8 units to (8). At 2 seconds you would have traveled 4 more units to (12). At 3 seconds you would have traveled 2 more units to (14) and so on.

So it looks like in one "second" you teleported 8 units and did not interact with any of the space between unit 0 and 8. But in the real world we know you can't teleport, and I assume that in that one second you would have visited every single space in between unit 0 and 8, whether that's an infinite amount of space, or a measurable amount of space at a sub-atomic level. Let's assume it's measurable, and that there is a corresponding unit of time that captures your movement without any teleportations called X.

But if that's the case then as you go from unit 8 to 12 in the same amount of time (one second), you're covering less space. So under that same measurement of time X, you'd see places that you wouldn't move, and wouldn't be at motion.

Or alternatively does it mean that when you go slower, you aren't changing your speed, but changing the value of X. And does that mean that time can be controlled by how fast you move.

>> No.15418621

>>15418598
Are you high?

>> No.15418967

[math]x=\log _2(8-2 x)\ [/math]

how ? I woke up retarded

>> No.15419025

>>15418967
Raise both sides to the power of 2:
[math]2^x=8-2x[/math]
Rearrange to put all the x's on one side:
[math]f(x)=2^x+2x=8[/math]
The function [math]f[/math] is differentiable and its derivative is always positive, so it is strictly increasing and [math]f(x)=8[/math] at exactly one point [math]x[/math]. Eyeballing the graph (or just trying random values for [math]x[/math] and using the strictly-increasing property of [math]f[/math] to adjust your guess higher or lower) suggests that this point is given by [math]x=2[/math], which you can confirm by substituting and evaluating [math]f(2)[/math].

>> No.15419033 [DELETED] 

>>15418967
>>15419025
>The function f is differentiable and its derivative is always positive, so it is strictly increasing and f(x)=8 at exactly one point x.
Addendum: Strictly speaking, you'd have to confirm that 8 is within the range of values for [math]f[/math]. Fortunately, the range is [math]2<f(x)<\infty[/math] (since [math]f(x)=2^x+2x[/math]), so we are safe in this case.

>> No.15419070

>>15419025
Seems odd you can only solve it by looking at the graph.

>> No.15419082

>>15419070
You generally can't solve any random combination of exponentials and polynomials analytically. Maybe if you can put it in a form for the Lambert W function you could.

>> No.15419522

I'm kind of having a hard time understanding rigid body physics, I am looking for a dumbed down explanation of the basics (possibly not a video but text, for example a student friendly textbook) that I'll read in addition to my textbook which already has all the formal definitions

All I understood so far is that there are quantities which are dual to the ones already defined (e.g. mass->moment of inertia, force->moment of the force (torque)) which accounts for the fact that a rigid body can also spin instead of just translating like a point.

>> No.15419640
File: 8 KB, 524x172, asdf.png [View same] [iqdb] [saucenao] [google]
15419640

How do I calculate [H+] given the molarity of my solution of NaHCO3 is 44 mM? How do I find [HCO3-]? [CO2(aq)] = 1.519*10^-3 mol/L

>> No.15419883

>>15419640
Don't you just look up in a table the acid dissociation constant of HCO3- and calculate it from that?

>> No.15420012

>>15418560
To add to that answer: (pseudo-)randomness is taken quite seriously in cryptography. There are instances of schemes where replacing a "true random generator" (a theoretical one if you will) by a pseudo-random generator, however good it is, lead to total collapse of the security of the scheme. See CGH98 for the article.

>> No.15420246

>>15412116
Can someone please help me find a formula for the nth element of each of these sequences? I will post the first several elements of each sequence below, and then explain how they are defined.

S_a: 3, 9, 15, 27, 33, 39, 45, 51, 57, 63, 75, etc
S_b: 3, 15, 27, 33, 45, 57, 63, 75, 87, 93, 105, 117, etc
S_c: 3, 33, 45, 63, 75, 87, 105, 123, 135, 147, etc
s_d: 3, 33, 63, 75, 105, 123, 135, 147, etc

The sequences are defined as follows:
S_a: All odd multiples of 3.
S_b: All members of S_a without a remainder of 1 or 4 when dividing by 5.
S_c: All members of S_b without a remainder of 1 or 6 when dividing by 7.
S_d: All members of S_c without a remainder of 1 or 10 when dividing by 11.

In general to get to the next sequence, you use all members of the previous sequence without a remainder of 1 or p-1, where p is the next prime number.

>> No.15420257

>>15420246
>S_a: All odd multiples of 3.
skipped 21

>> No.15420307

>>15420257
Oh yeah, 21 is supposed to be in there, thank you. I did those sequences by hand so there could be mistakes.

>> No.15420410

>>15412116
do you have that meme screenshot "how a scientist looks like" with a black woman with a funny hair. need for research purposes ofc

>> No.15420425
File: 5 KB, 374x337, 87474.png [View same] [iqdb] [saucenao] [google]
15420425

How do I make a parametrization [math]r(\varphi)[/math] of r?
I found this relation [math]r=\frac{2scos(\varphi)}{\pi-2\varphi}[/math] but this still depends of s (the arc length) so i might be doing something wrong.

>> No.15420488
File: 25 KB, 474x320, th-3827842290.jpg [View same] [iqdb] [saucenao] [google]
15420488

I am studying bones. Where can I find good images of real ones? Google rarely hits the mark.

>> No.15420644 [DELETED] 

>>15420425
I assume [math]\bar r[/math] is the length of the line segment? If so, I get [math]\bar r(\varphi) = r\sqrt{2-2\cos\varphi}[/math], where [math]r[/math] is the circle's radius.

>> No.15420663

>>15420425
I get [math]\bar r (\varphi) = 2r |\cos \varphi |[/math], where [math]r[/math] is the circle's radius and [math]\varphi[/math] is the angle as in your picture.

>> No.15420699
File: 215 KB, 1970x290, Problem.png [View same] [iqdb] [saucenao] [google]
15420699

Hello /sci/entists!

I've had trouble trying to understand this. Please, can somebody help me?

Thank you and have a nice day.

>> No.15420713

>>15420663
Thanks a lot anon! i kind think of that one cause i knew r(0) must be 2r and r(pi/2) must be 0, but wans't really sure how to prove that so i was maybe wrong.

>> No.15420719

>>15420699
The statement in the picture doesn't make a lot of sense to me, but I'm pretty sure you're meant to prove that [math]\sim[/math] is an equivalence relation, where we define [math]x\sim y[/math] when [math]x\in S\implies y\in S[/math] for some [math]S\in P[/math]. Basically the elements of the sets that make up the partition are said to be equivalent.

>> No.15420754

>>15412116
How do I get better at writing argumentative essays for AP lang? Sometimes I just dont have an opinion on the prompt at all and it worries me that there is only one prompt to choose from on the AP test

>> No.15420780
File: 194 KB, 1500x1500, FvcaCFTaYAA8rUP.jpg [View same] [iqdb] [saucenao] [google]
15420780

>>15419070
the expression isn't solvable analytically because its a combination of exponentials and linear terms.

You can, of course, use rood-finding approximation methods like newton's method to approximate the root of say [math]2^x+2x-8=0[/math], and with enough precision you can probbaly guess it converges to an integer solution and test it out by directly plugging it into your initial equation and verifying

>> No.15421061

>>15420754
why the fuck are you taking ap lang? take a writing course at city college, blow all the druggy fuckups out of the water, get straight A's and transfer credits

>> No.15421112

>>15420754
you dont, frq scores are literally random on ap lang I genuinely had no idea what I was doing and still got a 5 just go with your gut.

>> No.15421114

>>15421061
Because AP tests are less expensive than enrolling even in community college

>> No.15421118

>>15412116
In this here question, the first answer:
> https://math.stackexchange.com/questions/2128462/gradient-of-squared-frobenius-norm-of-a-matrix

I don't understand what happened at this step:
[math]\nabla_W(Tr(W^T X^T X W - Y^T X W - W^T X^T Y + Y^T Y)) = 2 X^T X W - 2 X^T Y. [/math]

Can someone explain? Those are all matrices with similar dimensions.

>> No.15422113
File: 58 KB, 1440x314, Screenshot_20230507-181228.jpg [View same] [iqdb] [saucenao] [google]
15422113

What the hell happened her

>> No.15422623

>>15422113
Use the substitution [math]x = u^2 - v^2, dx = 2u\ du[/math]

Then the inner integral becomes
[eqn]
\int \frac{uv}{\sqrt{u^2 - v^2}}\ du = \frac{v}{2} \int \dfrac{1}{\sqrt{x}}\ dx = \frac{v}{2}\ 2\sqrt{x} = v\ \sqrt{u^2 - v^2}
[/eqn]

>> No.15422692

>>15422623
Yes of course, I'm too busy thinking in multivariable that I forget to practice the methods I learned in one variable, thank you.

>> No.15422722

I'm trying to do these two exercices from A First Look at Rigorous Probability Theory by J.S. Rosenthal (exercise 2.2.3. and exercise 2.2.5). I'm very new to probability theory so I don't know how to do the proofs properly. Any anon can spoonfeed me?
>Consider the uniform [0,1] distribution. Define J as the collection containing all the open, closed, half-open, singleton and empty intervals in [0,1]. Prove that J is a semialgebra of subsets of [0,1], meaning that it contains [math]\emptyset[/math] and [0,1], is closed under finite intersection, and the complement of any element of J is equal to a finite disjoint union of elements of J.
>Prove that the set B of all finite unions of elements of J is an algebra of subsets of [0,1]. Prove that B is NOT a sigma-algebra.

>> No.15422773

>>15422722
>is closed under finite intersections
You can just show that it's closed under a single intersection (by exhaustion, basically) and extend by associativity.
>the complement of any element of J is equal to a finite disjoint union of elements of J.
This is obvious. Just exhaust on the possible cases and remember to account for the border.
>Prove that the set B of all finite unions of elements of J is an algebra of subsets of [0,1].
Intersection distributes over unions.
>Prove that B is NOT a sigma-algebra.
It doesn't contain i.e. [math]\{ 1/2^n : n \in \mathbb{N} \}[/math]

>> No.15422973

>>15421118
https://atmos.washington.edu/~dennis/MatrixCalculus.pdf

>> No.15423091

So are peer reviewed papers basically just the adult version of a classmates homework? You should expect lots of mistakes and terrible communication, right? I thought they were held to a higher standard.

>> No.15423595

>>15423091
>So are peer reviewed papers basically just the adult version of a classmates homework
Everything is the adult version of a classmate's homework
>You should expect lots of mistakes and terrible communication, right
Only if the reviewers didn't do their job, which is more likely in meme journals. Don't read meme journals. Or at least read them more critically.
>I thought they were held to a higher standard
Anyone can write a journal. Some journals are better than others at filtering the trash.

>> No.15423857

Hello /sci/autists,
How do I solve a seven or more variable system of inequalities like
a>0, b>0, c>0, d>0, e>0, f>0, g>0, 0.66a-b-c-d-e-f-g>0, 14b-a-c-d-e-f-g>0, 8c-a-b-d-e-f-g>0, 14d-a-b-c-e-f-g>0, 6.5e-a-b-c-d-f-g>0, 8.5f-a-b-c-d-e-g>0, 22g-a-b-c-d-e-f>0
I don't need the actual solution for the inequalities, just any set of 7 seven values that satisfies the system, or even just knowing if there are any solutions will do.
Wolfram and chatgpt (shut the fuck up) don't help, should I just run a montecarlo simulation? Thanks in advance.

>> No.15423985

>>15423857
Isn't this a linear programming thing?

>> No.15424196
File: 15 KB, 563x362, linprog.png [View same] [iqdb] [saucenao] [google]
15424196

>>15423985
It's not, because he has only strict inequalities. Linear programming needs the feasible region to be closed for the objective function to attain a maximum.
>>15423857
Looks like no solutions exist.

>> No.15424249

>>15424196
>It's not, because he has only strict inequalities. Linear programming needs the feasible region to be closed for the objective function to attain a maximum.
Does it really matter in this case?
If we set up a dummy function a+b+c+d+e+f+g to be maximized and lin prog finds a solution with weak inequalities then the only case where it would fail with strong inequalities is if the only solution whatsoever (regardless of maximum) was a single point, that is you replace all inequalities by being equal to zero. If your max solution differs from the "minimal" solution then it should imply the existence of a region where the strong inequality holds, barring cases where all the linear constraint point to exactly the same line.
If simplex moves from one vertex to another then that should be enough, and if we have 3 vertices then we could take a linear combination of them to obtain a sample solution to strong inequalities.
Unless I'm missing something

>> No.15424316

>>15412116
how do locks and keys work? what standards systems are used in the locksmithing industry, and where can i learn the basics of modern lock design?

>> No.15424365

>>15423857
There are no solutions. Add a-a, b-b, ... to each equation and substitute a+b+...+g=s:
1.66a>s, 15b>s, 9c>s, 15d>s, 7.5e>s, 9.5f>s, 23g>s
Divide each inequality by the constant coefficient:
a>s/1.66, b>s/15, c>s/9, d>s/15, e>s/7.5, f>s/9.5, g>s/23
Add them together and substitute a+b+...+g=s:
s>s(1/1.66+1/15+1/9+1/15+1/7.5+1/9.5+1/23)
=> s>1.1289s => 1>1.1289

>>15424249
> Does it really matter in this case?
No. You can just replace the > with ≥ and solve. If you get a solution that's not all-zero, the feasible region isn't empty. A solution of z=k for the modified system means z<k for the original system.

>> No.15424452

Why can't I make threads while on my phone
>inb4 kill yourself phoneposter

>> No.15424881

>>15420488
Get an anatomy atlas, if you're insistent on realistic images then search for "Anatomy: A Photographic Atlas" by Johannes W. Rohen, otherwise use Netter's Anatomy.

>> No.15425418
File: 144 KB, 1080x902, neil.png [View same] [iqdb] [saucenao] [google]
15425418

Scientifically speaking, there is no real point or meaning to life. We just kind of exist because all the variables were just right, but for all intents and purposes our existence if a mere coincidence. Which leads me to believe that life doesn't matter. Not morally or ethically, just scientifically. So scientifically speaking, black lives also don't matter. Right?

>> No.15425695

What program do I use to simulate a custom ADC? There's too many versions of spice, I don't know what to use. I also have access to synopsys and cadence tools

>> No.15425919

why cant a fridge freezer be used as storage like a battery? it functions the exact same
especially the ones which use gas/compression

>> No.15425929
File: 412 KB, 500x502, May7Phenomenon1.webm [View same] [iqdb] [saucenao] [google]
15425929

On May 5 and May 7 something very strange happened to every single weather radar. The storms appeared to be jumping back and forth.
There was also a CME on May 5 and May 7 that lines up with the radar phenomenon.
Does this prove that the magnetic field is weakened?
This video has good documentation of the May 5 incident.
https://youtu.be/AvmH2qL6F2A

>> No.15426002

I played the pass out game 1 time when I was a kid, did I get brain damage from it?

>> No.15426026

>>15425695
LTSpice is the free one
>b-b-but
use it and fuck off

>> No.15426063

>>15425929
It's the effects of La Nina. Cold air from the north is interacting with warm air from the Gulf of Mexico. It's why there are so many tornadoes in the midwest this time of year.

>> No.15426095

>>15425929
>>15426063
There was a massive heatwave which came up through Texas into Kansas during this period which was likely the exact reason the rain didn't make it south.

>> No.15426101
File: 30 KB, 474x531, th-1592245202.jpg [View same] [iqdb] [saucenao] [google]
15426101

Hey anons,what are you reading right now? On average, how long does it take you to complete a book and how many pages, assuming you also work on the exercise?

>> No.15426508

What is the libgen equivalent for audibooks?

>> No.15426509

>>15426508
youtube

>> No.15426735
File: 7 KB, 229x250, 1663421100961529.jpg [View same] [iqdb] [saucenao] [google]
15426735

>speed of light is 299792458m/s
Relative to what or whom? Whose seconds, since the length of one second for the observer is not the same as for the one travelling at c? If A is travelling at 0.5c along the x axis, and B is travelling at 1c along the x axis in the opposite direction, is B not travelling at 1.5c relative to A?
>yeah but only absolute speed matters
Ok, then what's the reference point? What's the absolute zero of speed compared to which c is the speed of light?

>> No.15426756

>>15426735
> since the length of one second for the observer is not the same
not true. the length of one second is the same for all observers in their reference frame.

> What's the absolute zero of speed
it doesn't exist. all velocities are relative, only acceleration is absolute.

>> No.15426774

>>15426735
>If A is travelling at 0.5c along the x axis, and B is travelling at 1c along the x axis in the opposite direction, is B not travelling at 1.5c relative to A?
No
https://en.wikipedia.org/wiki/Velocity-addition_formula#Special_relativity

>> No.15426908

>>15426774
>>15426756
>>15426735
nta but related, have we empirically validated the lorentz factor? I know we see relativistic effects on gps satellites, but has anything else had to take relativity into account?

>> No.15426912

>>15426908
There are literally thousands of experiments showing that relativity is a real effect. You can even derive it from Maxwell's equations and they were written down 40 years before special relativity.

>> No.15426918

>>15426912
>you can derive it from maxwell's equations
that literally not the definition of empirically. I'm not doubting relativity, I'm just asking if we've validated it fully. Maybe when you actually get to .8c, there is another unknown factor we aren't accounting for at .1c, and by then you get really inaccurate values. Thats all I'm asking

>> No.15426924

>>15426908
https://en.wikipedia.org/wiki/Tests_of_special_relativity

>> No.15426930

>>15426918
I have no idea what you are trying to say since the definition of empirically is "based on experience (experiment) rather than theory". It has been proven by both.

>> No.15427043

>>15424881
Already have Netter's atlas of human anatomy. It's very good, but lacks certain angles and details. For example the transverse process of t12 is kind of vague, had to do a lot of searching to get a good grasp of it. I'll look into the Rohen book, thanks.

>> No.15427157
File: 95 KB, 610x610, aquaman.jpg [View same] [iqdb] [saucenao] [google]
15427157

What is the benchmark for meaningful dechlorination of water?

I am a physicistlet and I'm told to BOIL water to use for stuff like some fermentation processes just so it can expunge the chlorine... Can't I just pour it from jug do jug over big distance or put it into a blender?

Right now I want to try keeping chlorine out of my hair and boiling that much (or having it sit for days) seems ridiculous, as is the wait.
>there's purifie--
poor

I just need a cup or two at a time for my buzzcut head, really

>> No.15427194

imagine for me that you are a person in your late fifties with hypertension. now, suddenly, your heart rate fucking jumps from 90 bpm to 170 bpm. what caused it?

>> No.15427236 [DELETED] 

>>15427194
Does:
[math] \forall t \in \mathbb R \quad
P( Y < t ) = P ( X \leq t) \implies \forall t \in \mathbb R \quad P ( Y \leq t) = P ( X \leq t) [/math] ?

>> No.15427238

Is it true:
[eqn] \forall t \in \mathbb R \; P( Y < t ) = P ( X \leq t) \implies \forall t \in \mathbb R \; P ( Y \leq t) = P ( X \leq t)
[/eqn]

>> No.15427259

>>15427157
Chlorine is volatile, so aeration is the simplest way to remove it. Pumping air through it or spraying it works to that end, though I don’t know how long/much it would take.
HOWEVER: If the water is treated with chloramine, aeration doesn’t work. Even boiling is largely ineffective. Only specific filtration methods are effective on it.

>> No.15427262

>>15427259
*or dechlorination chemicals (like for aquariums) but you probably don’t want to use that if you’re brewing with it.

>> No.15427271

Any suggestions of textbooks talking about integration theory? I'm reading a probability theory textbook and realizing I don't really understand what a Lebesgue integral really is. Something that is rigorous but at the same time also helps build intuition would be perfect.

>> No.15427310

>>15427259
>Chlorine is volatile, so aeration is the simplest way to remove it. Pumping air through it or spraying it
unnecessary. simply leaving a jug of water out for 30 minutes removes it all.

>> No.15427312

>>15427194
Have you gone off benzos? Diet change?

>> No.15427315

>>15427310
>removes it all.
What I heard is it requires days. Can you back this up?

>> No.15427319

>>15427194
doctor, immediately. you shouldn't see those rates unless you're jogging.

>> No.15427320

>>15427315
just based on taste tests and having it not killing all my bread yeast. chlorine is easy, but chloramine is the one that would take days.
if you're the scalp schizo take your meds.

>> No.15427336

>>15427320
>if you're the scalp schizo take your meds.
Look buddy, I'm just following mayo clinic/w/e articles and experimenting to make my fucking skin condition go away. One theory being chlorine unbalances the microflora, giving some microbes a head start.

Aren't you a bigger schizo? You go by taste as if it's somehow guaranteed your senses don't fall the fuck off the moment the chlorination falls by 8%.

>> No.15427357

hello /sqt/
I believe that if we have a countable sequence of infinite (possibly disjoint) sets which are all of the same cardinality then the sum is the cardinality of any single set, i.e. a countable sum of a set of sets whose cardinality is the same is equal to that cardinality
however I have not been able to prove it. Any help?

>> No.15427363

>>15427336
then buy a $1 jug of distilled water or a $10 brita filter
"unbalances the microflora" is complete pseudoscience btw

>> No.15427373
File: 29 KB, 500x500, soy crack.png [View same] [iqdb] [saucenao] [google]
15427373

>>15427363
>chlorine affecting microorganisms is hecking pseudoscience!

>> No.15427377

>>15415604
Very happy with it, and completely agree with the other anon that called it boring. On the contrary to Rudin which is considered dry by all, Zorich's book includes applications from physics, which is nice to remind the reader that the material is of course useful. There's a pretty fair MAA review here.
https://www.maa.org/press/maa-reviews/mathematical-analysis-i-0

>> No.15427380

>>15427373
no, you're right anon. the precious little glass bugs on your scalp are shitting themselves under a stiff breeze, that's why your scalp is red and covered in flakes

>> No.15427422

Let's say I'm making a solution, something like simple syrup (mixing sugar into hot water at a 1:1 ratio by volume)
how does the volume of the simple syrup compare to the volume of the water and syrup alone
if I put one cup of sugar into one cup of water, I don't just get 2 cups of syrup so what do I get? What is the relation between the volumes of solutes and solvents vs. the final solution volume, for solutions in general?

>> No.15427448

>can't play HLS in Firefox for some reason now
https://bpa.st/2ZFK6
which one of these broke it?

>> No.15427633

Prove 3x^2 - 2x + 1 is continuous at x = 2 using delta-epsilon.

I was doing the delta epsilon proof, and got |(3x+4)(x-2)| < epsilon.
I'm not sure where to go from here. I know that (3x+4) = 3(x-2) + 10, but I don't know how to use this in the epsilon proof.

>> No.15427701

>>15427633
You have to show that for every epsilon there exists a delta such that |x - 2| < delta => |f(x) - f(2)| < epsilon. This essentially means that for any epsilon there exists some delta around 2 where the function is close to it's value, as close as the epsilon is. Note that the delta depends on the epsilon.

>> No.15427900

Can you write (and publish) academic papers if you dont have a degree? Let's say some autistic shut-in just does math all days, could he also write papers?

>> No.15427905

>>15427900
Sure but without any institutional backing it would be harder for any paper to be reviewed and accepted.

>> No.15428043
File: 16 KB, 490x240, electron_decay-2224371216.gif [View same] [iqdb] [saucenao] [google]
15428043

If you could de-exite all the atoms in a given area would that reduce interference in QC?

>> No.15428067

>>15428043
All atoms are already in their ground state.

>> No.15428070

>>15428067
100% of the time?

>> No.15428079

>>15428070
Not 100% since you have thermal radiation and cosmic rays, but that only lowers it to ~99.99999999% of the time.

>> No.15428082

>>15428079
But trillions of atoms even at 99.99999999% it will still happen.

>> No.15428092

>>15428082
Yeah, that's what I just said but it's so tiny of an effect it can be ignored. All physical systems want to be in their lowest energy state and that includes atoms. Excited electrons will decay almost instantly. You have to actively pump energy into a system to counteract it.

>> No.15428101

>>15428092
Tiny enough to not ever collapse a waveform?

>> No.15428111

>>15428101
Are you talking about superpositions?

>> No.15428114
File: 84 KB, 719x500, I13-11-qbit-3118215408.jpg [View same] [iqdb] [saucenao] [google]
15428114

>>15428111
Qbits

>> No.15428120

>>15428114
*any* interaction with the environment will break a superposition. There's no perfect vacuum, they will touch the apparatus walls, etc. That's why qubits are so unstable.

>> No.15428124

>>15428120
Yeah I know that, what I was saying even if each atom has a 99.99999999% chance of them being in a ground state multiplied by the fact there are literally trillions of them the chances of them being in an excited state and the fact that there is a non-zero chance of them being found outide their orbitals, it will happen often and it COULD cause interference.

>> No.15428222

>>15428124
What makes you think one of those incredibly rare events is going to interact with a qubit? All those other possible scenarios happen far, far more frequently.

>> No.15428227

>>15428222
Not the qubit interaction with the surroundings/housing which then interacts with the qubit, i'm pretty sure the interference is coming from somewhere not accounted for by those.

>> No.15428235

>>15428227
You could be struck my a comet, but you're more likely to be struck by a car (or if you're in the US - a bullet).

>> No.15428255
File: 45 KB, 362x356, lasercooling.png [View same] [iqdb] [saucenao] [google]
15428255

>>15428235
True, anyway I will ponder to see if I can think of a way to keep an atom in its ground regardless of interaction.

>> No.15428382
File: 197 KB, 2562x266, image_2023-05-10_014808605.png [View same] [iqdb] [saucenao] [google]
15428382

This problem is from a sample final for my real analysis class. I'm not really sure how to approach it.

>> No.15428415

>>15428382
Hint. Consider two cases, [0,M+1] and [M, inf] where the function is close to its limit after M.

>> No.15428427

>>15413830
>all the laws of physics obey T symmetry
assuming CPT theorem, then CP-violation (which occurs in the weak sector) is equivalent to T-violation. so not all physical laws obey T symmetry, if we're being pedantic

>> No.15428445

>>15427422
https://en.wikipedia.org/wiki/Partial_molar_property

>> No.15428448

>>15428427
>then CP-violation (which occurs in the weak sector) is equivalent to T-violation
no it's not, when CP is violated T remains unaffected.

> so not all physical laws obey T symmetry
every physical law obeys T symmetry.

>> No.15428451

>>15428043
no. because as the other anon pointed out, you are not going to be sensitive to errors of that magnitude, given your other error sources are many orders of magnitude larger. the infrequency of non-ground-state atoms would be mitigated by quantum error correction after the fact, not by preventing them from occurring in the first place. just like how computers can account for errors due to cosmic rays, rather than preventing the rays from hitting the computer in the first place.

>> No.15428456

>>15428448
>when CP is violated T remains unaffected.
no. CP violation is the same thing as T violation, under the assumption of CPT symmetry.
https://en.wikipedia.org/wiki/CP_violation#Overview
if CP was violated without violation of T symmetry, it would imply CPT violation. which has never been observed.

>> No.15428466

>>15428451
Okay, thanks.

>> No.15428481
File: 773 KB, 902x2806, e06e071b7b76ff41226c05920fbc66579f1fa271.jpg [View same] [iqdb] [saucenao] [google]
15428481

Does subject reduction hold for a generic pure type system (not necessarily a member of the lambda cube)?
I've managed to reduce the problem to the base case [math]\Gamma \vdash (\lambda x:A.B)(t):C \implies \Gamma \vdash B[x:=t]:C[/math], and the usual recommended strategy is to induct on the structure of [math]B[/math], but I don't see how that helps prove even the simplest case [math]B=x[/math] without becoming circular.

>> No.15428510

>>15428456
You need to be careful here:
> a violation of the CP-symmetry is equivalent to a violation of the T-symmetry
this means mathematically it can be seen as equivalent but it not an actual observation of T symmetry violation.

Also the citation linked in that section is now out of date. More recent would be https://www.annualreviews.org/doi/10.1146/annurev-nucl-102014-022251 has a good collection of experimental tests.

Their conclusion:
> no signal has been observed to date, upgraded and new experiments in the next decade will increase their sensitivity by approximately two orders of magnitude.

So far, still no violation.

>> No.15428533

>>15428510
Okay, I agree with what you're saying in the sense that "no experiments have directly observed T-violation."
However, the development of the universe and "arrow of time" imply T-symmetry violation. We just don't have the formalism to describe it from first principles.
In addition, CP violation does theoretically lead to T-violation, in a mathematical sense but also in a physical sense. CP-violating processes would lead to an electron having an electric dipole moment via the CKM matrix, but the nature of this moment is that it is T-violating.

>> No.15428552

>>15428533
I just can't wrapped my head around the idea of a system that isn't time reversible which is why I'd need to see definitive proof.

>> No.15428633

Does anyone know of a really good undergrad level summary of waves, light and optics courses?

>> No.15428647

>>15412116
what's the minimum amount of rain it takes to make the road wet?
like, not wet wet, just all moist I guess
is it even 0.1mm?

>> No.15428699

Would really appreciate it if I could get some help for this problem from my Real Analysis Class:
Let f (x) be a differentiable function over [0, 1]. Suppose that f(0) = f(1) = 0. Prove that f′(x) − 2f(x) must have a zero inside (0, 1). Namely, there is a c ∈ (0,1) such that f′(c) − 2f(c) = 0.

>> No.15428744

>>15428699
Consider the function g(x) = f(x)*e^(-2x)

>> No.15428753

>>15428744
Why would you consider g(x) = f(x) * e^(-2x)?

>> No.15428756

>>15428753
It was a hint. Think about it for a while

>> No.15428757

>>15428699
Im currently taking real analysis too, so take my suggestion with a grain of salt. I think you can try IVT and try to bound some function st it has the following form (neg < g(x) < positive). Thus, there must exist c in interval where g(x) = 0.

>> No.15428765

>>15428756
I know you can use Rolle's theorem once you have this to prove the it to be true, but I still don't get why you use g(x) = f(x) * (e^-2x)
Where does the e^-2x come from and why?

>> No.15428781

>>15428765
It comes from the fact that g'(x) = e^(-2x)*( f'(x) - 2f(x) ) and the fact that the thing in the parentheses is the thing you want to find a zero of. This is similar to using integration factors when you want to solve a differential equation:
https://en.wikipedia.org/wiki/Integrating_factor#Solving_first_order_linear_ordinary_differential_equations

>> No.15428786

>>15428781
So if the question asked if f'(x) + cf(x) has a zero, g(x) = f(x) * (e^cx)?

>> No.15428791

>>15428786
Yes

>> No.15428797

>>15428791
Thanks I have a question about a similar problem:
Let f (x) be a differentiable function over [0, 1]. Suppose that f (−1) =1, f (1) = −1. Prove that there is c ∈ (0, 1) such that f (c) = −cf ′(c).
In this problem, you would have g(x) = e^c * f(x), but I'm not sure where to go from here. I think it has to be something like proving that f(0) > 0 and then use the mean value theorem, but I'm not sure.

>> No.15428800

>>15428797
*e^cx

>> No.15428813

>>15428797
You wouldn't use g(x) = e^cx * f(x) because
1. You don't know what c is here a priori
2. The c is multiplying f' and not f.
Instead, you can consider g(x) = x*f(x) here

>> No.15428819

>>15428813
Okay, so I get g'(x) = x * f'(x) +f(x)
Evaluating g at -1, 1 and 0 I get
g(1) = f'(1) - 1
g(-1) = -f'(-1) + 1
g(0) = f(0)
I'm not sure how to get rid of the f'(x) here.

>> No.15428824

>>15428819
I'm sorry my brain is really fried here this is a bad question please ignore it.

>> No.15428825

>>15428819
I don't see why you're considering g(0). g(1) = g(-1) = -1 here so you just apply Rolle's theorem to find a c in (-1, 1) such that g'(c) = 0

>> No.15428827

>>15428824
Too late, I already replied

>> No.15428833

>>15428827
Sorry :(, I've been up all night studying for my final and my brain is starting to fail.
Just to make sure I have everything right.
g'(x) = x * f'(x) + f(x)
g'(c) = 0 in [-1,1]
g'(c) = c * f'(c) + f(c) = 0
I'm not sure how to prove this lies in the interval [0,1].

>> No.15428837

>>15428833
I don't think it has to lie in [0, 1]. For something like f(x) = -x^3 (which satisfies the conditions of your problem), the c you get will be zero. So by perturbing f slightly, you should be able to move solution c to either the left or right of zero

>> No.15428843

>>15428837
I'll take your word for it but the question asks if there is specifically a c ∈ (0, 1) such that f (c) = −cf ′(c).
Another question, how did you get x * f(x) is it related to ODEs again or some other type of knowledge?

>> No.15428855

>>15428843
It's actually the same thing. Using the method of finding the integrating factor given in the link I posted earlier gives it to be [math] e^{\int \frac{1}{x} } = x [/math].

>> No.15428878

>>15428855
Sorry for asking this. In the case of f'(x) - 2f(x) = 0, the IF was e^-2x, which makes sense to me.
In the case of cf'(x) - 2f(x) = 0, wouldn't the IF be e^x/c? I'm confused on how that results in x.

>> No.15428900

>>15428878
In >>15428797, you want a c such that f(c) = -c*f'(c) i.e. a solution 'c' to the equation f(x) + x*f'(x) = 0. It makes sense that the IF should be x because if you define g(x) = x*f(x), then g'(x) = f(x) + x*f'(x)

>> No.15428910

>>15428900
ic, that makes sense
Also thank you so much >>15428855 for taking the time to help me through this stuff.

>> No.15428916

>>15428910
You're welcome, anon. I haven't posted helpful posts on /sci/ in a while

>> No.15429130

Can someone give me a quick rundown of the long-term effects of nicotine (such as from snuff or chewing tobacco, not smoking) on the cardiovascular system?

>> No.15429155

>>15429130
https://pubmed.ncbi.nlm.nih.gov/

>> No.15429344
File: 1.12 MB, 800x6100, leftist1.jpg [View same] [iqdb] [saucenao] [google]
15429344

>>15412116
cool article about scientists finally complaining about all the polticization see 6th article in that stack pages 40-65

>> No.15429354

>>15429344
>>>/pol/

>> No.15429658

>>15412116
how would i calculate the force this wheel exerted?

https://old.reddit.com/r/interestingasfuck/comments/123jat8/car_launched_into_the_air_after_a_wheel_detach/

is this a moment of inertia problem + net force( i suck at physics but im trying to improve my thinking by using real world situations.)

>> No.15429728

>>15429658
>how would i calculate the force this wheel exerted?
Do you mean the force when it hit the second car (not on the original car the wheel was attached to)? The force it exerts in the collision is something that changes rapidly during the brief moment of the collision, but you can calculate a time average of the force by finding the change in momentum of the car immediately before and after the collision (this is called "impulse" in physics 101).

>> No.15429786

>>15429728
yeah when it hits the second car. I for some reason forgot about impulse. Like i said im a dummy when it comes to physics even though the problems fascinate me. Also i noticed that when the wheel comes off it briefly accelerates faster so i would have to add on that speed in addition to what it was going at right before it came off (this is in los Angeles so the speed would be around 80mph/129 kmh).

>> No.15429795
File: 43 KB, 1000x500, reddit-discord-groomers-stonetoss-political-cartoon.png [View same] [iqdb] [saucenao] [google]
15429795

>>15429658

>> No.15429796
File: 54 KB, 750x731, 1872451027.jpg [View same] [iqdb] [saucenao] [google]
15429796

How do I know if I'm dumb or just tired? Sometimes I perform better than anyone and sometimes I perform horribly in everything. I always assumed that my mental abilities are bad but everyday I meet a new imbecile who proves to me that I'm not really dumb and I don't know anymore.
I lack any method to test if I'm actually intelligent or just incredibly lucky at times and IQ tests range from 110 - 160 which is bullshit IMO.
Am I just taking the wrong tests or is there truly no way to know?

>> No.15429800

>>15429795
yes i know no-one likes reddit but man is it that hard to keep an open mind and at least try to stay relevant to the topic?

>> No.15429986

Should I quote Nick Land in my thesis epigraph

>> No.15429996

>>15429796
IQ is a meme. Get into good sleeping/eating/exercise habits and you'll find that your performance is more stable at a reasonably high level. (ofc you will still have good days and bad days from time to time)

>> No.15429997

>>15429996
Art is the adult form of science. Why do we do science? If progression, then what for? Ultimately it is to become good artists and then produce pleasurable (or painful) sense data.

>> No.15430004

eww a tripfag just responded to my post. Will I ever be clean again?

>> No.15430054
File: 1.44 MB, 1200x1600, __houjou_satoko_higurashi_no_naku_koro_ni_drawn_by_3105lave__69ad86e8a986c6595539ca4abd41b1f9.jpg [View same] [iqdb] [saucenao] [google]
15430054

>>15418347
>linear algebra
https://www.youtube.com/watch?v=kjBOesZCoqc&list=PL0-GT3co4r2y2YErbmuJw2L5tW4Ew2O5B
>>15424316
https://www.youtube.com/watch?v=T_sy3dLwHkc
>>15425919
i suppose there isnt anything fundamentally keeping you from using the temperature gradient that a freeze creates as a battery, but it is very retarded.
>>15427357
im not an analysis expert but i dont see why you couldnt use induction here.
>>15427448
>user_pref("webgl.disabled", true);
>>15428633
openstax
>>15429986
depends. if its the "echos like a tomb" quote everyones gonna think youre a pseud.

>> No.15430113
File: 40 KB, 374x374, D8CRtMS.jpg [View same] [iqdb] [saucenao] [google]
15430113

"A particle of mass m moves under the action of the potential V(r) = −k/r, with k > 0.
In the case of bounded motion, if at t = 0, φ = 0 and r takes its minimum value, find r(φ) by solving the differential equation of the orbit,
[math] u^{\prime 2}+u^2+\frac{2 m}{l^2} V\left(\frac{1}{u}\right)=\frac{2 m E}{l^2} [/math]
You should find the differential formulation of a harmonic oscillator. The solution is
[math] r(\varphi) =\frac{1}{1+e \cos \varphi} \frac{l^2}{m k}[/math]
and
[math] e =\sqrt{1+\frac{2 E l^2}{m k^2}} .[/math]"
I don't get it, doing the change of variables u=1/r, then V(1/u)=-ku, so I got
[math] u^{\prime 2}+u^2-\frac{2 m}{l^2} k u=\frac{2 m E}{l^2} [/math]
and this doesn't seem like an harmonic oscillator. Am I missing something?
I thought about taking that equation derivative (basically, using binet equation) but tbf i think they wouldn't have told us to use specifically the differential equation of the orbit in that case (and I didn't get the differential formulation of a harmonic oscillator using Binet equation either lol).

>> No.15430148

How possible is it for someone whose probably closer to midwit then genius to understand enough math and physics to know where the fuck scientists are coming from when they say shit like "The universe is made up of strings" or "It's a 4D hypersphere" or "It's all electromagnetism bro"

Is it worth me trying to learn? I'm smart enough to realize that everyone around me is retarded, but also that I probably am too. And math was never my strong subject.

>> No.15430249

>>15430113
complete the square in the u^2-... u part of the equation and you'll get a shifted u variable and a correction to the energy term on the right hand side. Since you have a derivative of u squared plus u^2 It is like the kinetic energy plus potential energy of a harmonic oscillator.

>> No.15430254
File: 377 KB, 1364x1539, powder vs liquid.jpg [View same] [iqdb] [saucenao] [google]
15430254

I don't understand liquid laundry detergent, why do you need so little compared to powder?

Picrel. Same brand, same product line, just powder vs lquid.
For a
>4.5kg load
>normal soiling
>hardwater area
you need 112ml of powder, but only 40ml of liquid.
And let's be honest, the liquid is mostly water. How much water is it, like 80%? Let's be generous and say it's only 60% water. So once you remove the water it's 40%*40ml actual detergent.
Meaning you need 112ml of powder but only 16ml of the detergent in liquid to wash the clothes.

How does that make sense? Am I missing something?

>> No.15430379
File: 1.88 MB, 1403x1758, __houjou_satoko_higurashi_no_naku_koro_ni_drawn_by_hazumi_otoya__8ac302268062e3f9d4660b24d0addc64.jpg [View same] [iqdb] [saucenao] [google]
15430379

>>15430254
>the liquid is mostly water
not true. the reason the whole tide pod craze was so dangerous was because the shit was extremely concentrated. kids went to the hospital just from getting the stuff in their mouths, very caustic. if anything, the powder may be less concentrated.

>> No.15430393

>>15430379
this. washing power isn't really powder. it's more air filled granules so is much less dense / concentrated than the liquid form.

>> No.15430541

>>15412116
Are Thermal Plasma and Equilibrium Plasma the same thing? I'm doing my masters' on the nonthermal stuff but every paper calls them pretty differently

>> No.15431061

>Exercise: prove that a sentence written only with => cannot be logically equivalent to [math]\land[/math]
My proof is as follows. Let [math]\phi[/math] be such a sentence. Then, it has to consist of [math]\mathcal{A} => \mathcal{B}[/math] where those are also sentences.
But then you can write the truth table for those sentences, and [math] \mathcal{A} \equiv T[/math] while [math] \mathcal{B} \equiv \land[/math]. Therefore, we have infinite recursion or something and that seems like it's a contradiction.
Is this a proper line of reasoning? I am not convinced myself; it appears true (infinite recursion cannot happen in a finite sentence) but I can't think of a logical reason to actually say it's a contradiction.

>> No.15431064

>>15430541
https://en.wikipedia.org/wiki/Nonthermal_plasma

>> No.15431100

>>15431061
You've got the basic idea, but to turn it into a rigorous logical proof you'll need to appeal to something like structural induction.
Here's one way to go about it: let [math]X[/math] be the rightmost variable of [math]\phi[/math]: more precisely, if [math]\phi[/math] is atomic then we take [math]X=\phi[/math], otherwise [math]\phi = \mathcal{A} \Rightarrow \mathcal{B}[/math] and we take [math]X[/math] to be the rightmost variable of [math]\mathcal{B}[/math].
Then you can prove by structural induction that [math]X\Rightarrow \phi[/math] is always a tautology, which contradicts [math]X \nRightarrow X\wedge Y[/math] for any other variable [math]Y[/math].

>> No.15431283 [DELETED] 

My first Latex, hope it looks right (preview doesn't work for me dunno why):
\mathcal{X} = \mathbb{R}x\mathbb{R}, \left ( \left ( a,b\right ), \left ( c,d \right )\right ) \in \mathcal{R} if and only if b = d

Trying to figure out if this is an equivalence relation. I did some examples with only one pair in brackets, but how does it work when there's two? Is there a video or something you guys recommend I could watch on this?

>> No.15431284

My first Latex, hope it looks right (preview doesn't work for me dunno why):
[math]
\mathcal{X} = \mathbb{R}x\mathbb{R}, \left ( \left ( a,b\right ), \left ( c,d \right )\right ) \in \mathcal{R} if and only if b = d
[/math]
Trying to figure out if this is an equivalence relation. I did some examples with only one pair in brackets, but how does it work when there's two? Is there a video or something you guys recommend I could watch on this?

>> No.15431485

I'm >>15430113, in this same problem I'm asked to find the period of the orbit using the differential equation given by the energy. I was able to write dt= something dr. I guess the limits of the integral in t are from o to T (being T the period of the orbit) but which limits am I supposed to use in r?
>>15430249
thanks man, after posting i realize i could complete squares but still wasn't sure about the integral answer, now is pretty clear.

>> No.15431508

>>15431283
[math] \mathcal{X} = \mathbb{R}x\mathbb{R}, \left ( \left ( a,b\right ), \left ( c,d \right )\right ) \in \mathcal{R} [/math] if and only if [math] b=d [/math]

>> No.15431516
File: 23 KB, 565x77, Screenshot 2023-05-11 22.38.03.png [View same] [iqdb] [saucenao] [google]
15431516

>>15431284

have a look by yourself

>> No.15431549

Should we subject research papers to the death of the author viewpoint?

>> No.15431594

>>15431284
Yes, it's an equivalence relation. It's quite easy to see why - e.g. (x,1) is one equivalence class (where x is in R).

>> No.15431984

Why can we distinguish the smell of a dead human body from other animals but not the smell of something like a raccoon from a deer?

>> No.15432517
File: 153 KB, 1080x853, Screenshot_20230511_232303_Firefox Nightly.jpg [View same] [iqdb] [saucenao] [google]
15432517

>>15412116
Help please...

>> No.15432527

Is a virus that turns people into monsters like in Dead Cells actually possible?

>> No.15432607

>>15430054
I'm not sure what it has to do with analysis or how you could use induction here, and the problem is actually more subtle than it appears (depends on choice), see for example https://math.stackexchange.com/questions/249323/countable-unions-of-countable-sets

>> No.15432706

>>15430254
Yes you are missing the two most important things. You are not accounting for cost or cleaning ability. They have somewhat fixed the cleaning ability problem with multiformula pods with seperate spaces to prevent reaction. Powder just uses caking agents to prevent reaction.
But this only made the cost problem worse, and pods still arent as good as powder through all temperatures and water types, always use powder.
Unless you have thing for spending more per load with less cleaning ability.
Pods are literally tiny factory spoilers on a civic, increases cost, adds weight and non-functional drag, but for women (sparkles and weird shit they can talk about).

>> No.15432994 [DELETED] 

What's the most chemically inert porous and readily available material I can use for soaking in water and refreshing cigar humidor bags with? I currently use a piece of broken terracotta pot but after a few years it passes a slight flavor onto the humidor bags.

>> No.15432995
File: 189 KB, 271x300, 1655271900136397.png [View same] [iqdb] [saucenao] [google]
15432995

What's the most chemically inert porous and readily available material I can use for soaking in water and refreshing cigar humidor bags with? I currently use a piece of broken terracotta pot but after a few years it passes a slight flavor onto the humidor bags.
Sorry if its more of a /diy/ question but its going in my lungs so I'd much rather ask /sci/

>> No.15433036

>>15427238
Yes. [math]P(Y=t) = \lim_{s\downarrow t} P(Y< s) - P(Y< t) = \lim_{s\downarrow t} P(X\leq s) - P(X\leq t) = P(X\leq t) - P(X\leq t) = 0[/math] as a CDF is right-continuous.
>>15427271
I like Rene Schilling's MIMS for measure and probability theory, but I would expect a book on probability to explain what a lebesgue integral is in any case desu.

>> No.15433061

>>15433036
Sankyu anon-kun.

>> No.15434055

>>15432706
Well I'm not talking about pods (I know they're a gimmick and are needlessly expensive because of the novelty and 'convenience'. Plus I want fine tuning of dosing, I need to do loads of all sizes. To me, fixed pod doses are the opposite of convenient. Then again I'm not a dumb normie.).

I'm talking about regular liquid in a bottle.

But what you say is very interesting anyway - so normally, all the ingredients together would react? So both powder is filled with filler too? The extra bulk caking agent in the powder, and the water in the liquid bottle, are fillers that prevent premature reaction and keep the products stable?

>> No.15434426

>>15432995
>after a few years
brother just buy new bags...
I don't really know what "refreshing" entails but my best answer would be to go for a material that you can clean chemically after some time.

>> No.15434434

>>15412142
crescent is absolutely a fair and obvious synonym for increasing, you esl ape

>> No.15434693

Is the universe a real life example of an open set? You're always surrounded by more universe no matter where you are.

>> No.15434924

>>15434693
It is unknown.
If you're looking for "real life" examples of open sets you are just going to run into philosophical questions without much reality to them.

>> No.15435250
File: 78 KB, 894x901, 1670777372743729.jpg [View same] [iqdb] [saucenao] [google]
15435250

>>15434426
>I don't really know what "refreshing" entails
I'm re-hydrating cigar humidor bags with distilled water, pretty shrimple.
All it entails is putting the humidor bag in a tupperware bin with a porous object holding water for a few days, and a little heat.
>Boveda packs are made of a patented membrane that allows for purified water vapor to be emitted and absorbed continually. Inside each pack is a mixture of salt and purified water. Moisture is released through osmosis to achieve a predetermined RH in the environment the Boveda pack is placed in.
>after a few years
>brother just buy new bags...
They last 2-3 months if you don't re-humidify them. The only reason they stop working is because the moisture pack dries out. They're cheap but I'm conscious of limiting the amount of plastic consumables in my life as much as possible. Nothing wrong with what I'm doing, pretty much every cigar and pipe smoker does it. Now does anyone know the answer to my question?

>> No.15435269

>my best answer would be to go for a material that you can clean chemically after some time.
yeah exactly, i just can't think of a good material.
it either has to be something easy to clean or something more eco friendly to use as a consumable than plastic. terracotta pot hits all the marks except it imparts slight flavor. i'm using it for really heady weed and expensive cigars so even a slight flavor like that effectively turns it all into mids.

>> No.15435284
File: 429 KB, 1613x2055, PXL_20230513_115742569.jpg [View same] [iqdb] [saucenao] [google]
15435284

Say i have a pulley (see the pic) and the mass is sliding down (the pulley has its own mass too, if it matters)

Does the angular velocity of the pulley change if I suddenly remove the mass? Assume no friction whatsoever.

>> No.15435314

>>15435284
Removing the mass would mean the tension instantly becomes zero. Hence the (angular) acceleration would be zero and so the (angular) velocity would remain constant.

>> No.15435343

I have some basis [math]\epsilon_1, \epsilon_1 + \epsilon_2, -\epsilon_3[/math] in [math]\mathbb{R}^3[/math]. How do I check if it's oriented the same way as the standard basis?
By definition, I'd have to check whether the matrix containing the vectors of the basis as columns has a determinant > 0 but I don't know the vectors of the basis.

>> No.15435356

>>15435343
surely you would write them in terms of the standard basis you are checking against (if they aren't already in your example), no?

>> No.15435359
File: 2.38 MB, 1920x2560, 1683983328755.jpg [View same] [iqdb] [saucenao] [google]
15435359

Ok, I need to solder some wired onto some motors, however I didn't check for the terminals and now I notice they are female terminals, which completely fuck the soldering process. Pic related, wtf do I do?

>> No.15435378
File: 143 KB, 800x800, __remilia_scarlet_touhou_drawn_by_cato_monocatienus__3f7f7df32216145b65dd262d6a73268c.jpg [View same] [iqdb] [saucenao] [google]
15435378

>>15434434
Shut up larper, it's such a fucking archaism that Cambridge doesn't even list that usage of crescent and you'll really only get it if you're familiar with EML or speak some language that still uses that latin root.
https://dictionary.cambridge.org/us/dictionary/english/crescent

>> No.15435394
File: 101 KB, 501x464, Screenshot 2022-11-27 11.43.59 AM.png [View same] [iqdb] [saucenao] [google]
15435394

>>15412116
I've been trying to get an answer to this question for awhile, but nobody seems interested in providing a genuine, good faith response. I've already made a few threads on the topic, and most of the response are either from other people who are also interested in the question, or from butthurt normies and SJWs telling me to go back to pol.

I just want to know why the Lab Leak theory was considered a "far-right" conspiracy theory, that was supposedly being pushed by anti-vaxxers, white nationalists, and "The Russians".

See my post
>>15431940
>>15431940
>>15431940

>> No.15435398

>>15435394
Because most people pushing it *are* anti-vaxxers and/or white nationalists.

>> No.15435407

>>15435398
You're making that claim without evidence. Moreover, even if it is the case that the majority of these people are geuninely "white nationalists" (they aren't), that still would not logically imply that the Lab Leak Hhypothesis is false or that it is some sort of crazy far-right "conspiracy theory." This is an example of one of the most basic logical fallacies - the ad hominem.

>> No.15435411

>>15435394
>>15435407
All evidence during and after the lockdown showed it was not lab made. The vast majority of people pushing the storyline it was lab-made during and after the lockdown were anti-vaxxers or anti-china. The reason the lab exists in that location in the first place is precisely because it's one of the biggest hotspots in the world for new viruses appearing. If you want to read a good overview - https://www.reddit.com/r/science/comments/gk6y95/covid19_did_not_come_from_the_wuhan_institute_of/

>> No.15435415

>>15435407
I didn't say that. I don't know where the virus came from and I'm sure scientists are trying to figure it out. Maybe there is evidence to suggest it came from a lab in China, maybe not. The point is that any genuine arguments are drowned out by retards, and thus these arguments are associated with them. I am arguing using empirical evidence as a guide; demanding anything more is not realistic.
Also, pointing out logical fallacies is already very cringe, notwithstanding the fact that it isn't an ad hominem.

>> No.15435416

>>15412116
Anime is gay and for mentally challenged gays. Scientific fact backed by 1000's of studies.

>> No.15435427
File: 478 KB, 1695x713, 15427665546.jpg [View same] [iqdb] [saucenao] [google]
15435427

>>15435378
I'm a native speaker, you ridiculous turd. No one gives a fuck what your wikipedia-tier online dictionary says or doesn't say.

>> No.15435478

>>15406337
You now want to show [math]a f(a) + \int_{f(a)}^b f^{-1}(y)dy \geq ab[/math].
Notice that if [math]y \geq f(a), f^{-1}(y) \geq a[/math], so that [math]a f(a) + \int_{f(a)}^b f^{-1}(y)dy \geq a f(a) + a(b-f(a))[/math], as desired.
>>15435427
I have never seen crescent used to mean increasing (in a math context), but I did immediately understand what you meant.

>> No.15435535

>>15435478
I'm not the guy who used the word. I'm just pointing out that it's not bad English.

>> No.15435662

>>15435314
Tension becomes zero implies that the acceleration becomes zero because at that point the only forces on the system are the weight of the pulley and whatever force is keeping it from falling down which must be balanced since the pulley is not falling down, right?

>> No.15435676

>>15435416
get a job, nigger

>> No.15435706

>>15435662
No tension doesn't mean negative acceleration. Once you remove the weight the pulley is already rotating, it has momentum. In the ideal case with zero friction there is nothing to slow it down.

>> No.15435748

I'm trying to prove that
>[math] A \cap B = \emptyset[/math] iff [math]A\subset B^c[/math].
I did the proof in one direction, but for the other direction I'm trying to do proof by contrapositive and I'm having trouble.
What is the negation of [math] A\cap B = \emptyset [/math] and [math] A \subset B^c[/math]?

>> No.15435766

>>15435748
There exists x, x in A and x in B
There exists x in A, x not in B^C.

>> No.15435769

Did I fuck up by getting into Brilliant only now?
I'm a braindamaged idiot with a nearly completely atrophied quantitative brain muscle. I got it to improve and do a little math daily, but seems like they gimped the service and removed all community content that people loved.
It's too late for me to get a refund, but what do you think of Brilliant these days?

>> No.15435771

>>15435535
Even if it is not bad english, why make it harder for esls to understand the question?

>> No.15435820

>>15435771
I didn't write the question. It's obvious that whoever did write the question isn't a native English speaker. They were given incorrect advice about English and probably assumed the advice was correct. This is a thread that's supposed to be helpful, not troll people with bad English advice.

>> No.15435959
File: 437 KB, 1x1, An appeal for an objective, open, and transparent scientific debate about the origin of SARS-CoV-2 - The Lancet.pdf [View same] [iqdb] [saucenao] [google]
15435959

>>15435411
>>15435415

>All evidence during and after the lockdown showed it was not lab made
First of all that is not true - from the beginning their were mainstream scientists suggesting the possibility of a lab leak. In fact, Anthony Fauci was one of the people who thought it was a lab leak. Secondly, that is beside the point. The normies claimed that it was "settled science". That was literally the language that was being used. "Settled science". That we were completely, 100% certain it did not originate in a lab, and anyone claiming that it did was therefore an anti-science white trash schizophrenic loser. Moreover, the narrative in both the mainstream media and SJW circles was that any discussion of the topic was morally repugnant and that discussion of the topic should be censored on social media. In fact, many people were arguing that disinformation is a form of "domestic terrorism" and that people discussing stuff like the lab leak hypothesis or people who were critical of lockdowns should be criminally charged for spreading disinformation. By that very same standard, the pro-censorship types claiming that it was a "conspiracy theory" should be censored online (and ideally, we should pass laws to put the in jail). What is it that makes SJW disinformation different? In other words, when SJW's falsely claimed that the Lab Leak Hypothesis was a "racist conspiracy theory", that constitutes a form of disinformation, so how come what these individuals were doing is not "anti-science", and how come they should not be subject to censorship?

How come the Lab Leak Hypothesis is an "anti-science conspiracy theory", but denial of the Lab Leak Hypothesis is considered an "Anti-science conspiracy theory" by woke normies and the mainstream media, despite the fact that mainstream scientists maintain that the Lab Leak Hypothesis is a legitimate scientific question?

>> No.15435964
File: 76 KB, 1x1, The covid-19 lab leak hypothesis did the media fall victim to a misinformation campaign The British Medical Journal.pdf [View same] [iqdb] [saucenao] [google]
15435964

>>15435411
>>15435415
>>15435959 (cont.)

Admittedly, your Reddit link gives me the impression that you're just trolling and not entirely serious, but again that is exactly the problem. It is very difficult to find anyone from the anti-lab leak side who is willing to honestly discuss the topic, despite the fact that they constitute the dominant position on the topic in the media and pop culture.

Again, the question is not whether the Lab Leak Hypothesis is true or whether or not the majority of people talking about it are racist. The question at hand is an epistemological question and perhaps a linguistic question. I'm not asking a biological question about the origins of COVID, nor am I asking a sociological question about whether or not most people who believe the lab leak hypothesis are racists or conservatives. Once again, the question I am asking is what is it that makes the lab leak hypothesis an "anti-science" or "conspiratorial" position, whereas denial of the lab leak hypothesis is not "anti-science"? We here constantly in the media and the news that disinformation and science denial is not just a political concern, but a serious threat to the very foundations of democracy and modern civilization. If that is indeed the case, then why should we not be concerned about people who falsely claimed, without evidence, that COVID did not leak from lab? If the Lab Leak Hypothesis is indeed a legitimate scientific question - as you yourself seem to recognize on some level, since you acknowledge that scientists are currently investigating the topic - then falsely claiming that the Lab Leak Hypothesis is a "conspiracy theory" would itself constitute a form of science denial. What makes that kind of science denial more acceptable? Why shouldn't we be trying to censor people for calling the Lab Leak Hypothesis a conspiracy theory?

>> No.15436042

>>15435959
>>15435964
I'm not any of the posters you've interacted with. You're asking why politicians are inconsistent in the way they apply words, concepts, and rules? It's because they gain more from hypocrisy than they lose. You might enjoy reading some vintage Mark Twain if this sort of thing is where your thoughts are at the moment.

>> No.15436135

>>15435959
>>15435964
You are replying to two people. I did not post the leddit link though what was posted there did seem like a fine source of information.
Your two pdfs are lot more interesting than the twitter screenshot (lol) so I will have a read and think for a little while before I respond. Mind this thread archiving.

As for your comments:
>Once again, the question I am asking is what is it that makes the lab leak hypothesis an "anti-science" or "conspiratorial" position, whereas denial of the lab leak hypothesis is not "anti-science"?
Right now I am still of the opinion that the majority-held opinion by scientists is "not man made", and so rejection of a less likely hypothesis shouldn't be anti-science. On the other end, a lot of "lab leak" supporters are not arguing from a point of science, and I think a condemning response from the media (i.e. calling them anti-science) was/is justified.

>> No.15436572

does baking soda neutralize or degrade BPA? Not asking for a friend.

>> No.15436819

can someone make another thread? thank you.

>> No.15436830

i NEED HELP understanding the proof for a^2 < b^2 assuming 0<a<b

we use the given a<b to come up with a^2<ab and b^2>ab. then it follows a^2<ab<b^2 and thats the proof. But is it proven becuase we came up with another given using the inital giben that a<b? Is proving shit just using givens and conclusions to come up with other givens and conclusions??

>> No.15436835

>>15436830
not the proof but just proving shit in general

>> No.15436996

>>15435964
What do you mean by lab leak? All that literally means is that there is a chance a sample from an *already existing virus found in the wild* escaped. The evidence suggests the source was the Wuhan market, just like in previous events, but is it theoretically possible? Sure, but so what? Frankly it makes not a bit of difference.

>> No.15437018

>>15436830
>>15436835
>Is proving shit just using givens and conclusions to come up with other givens and conclusions??
Basically yes, however you will also need rules that tell you how the givens combine together to produce new conclusions.
For your example in particular, you are implicitly making use of a rule that says that if you are given 0<x and a<b, then you can combine these together (using multiplication) to get the conclusion ax<bx.